수반 함자: 두 판 사이의 차이

내용 삭제됨 내용 추가됨
잔글편집 요약 없음
135번째 줄:
* {{nlab|id=adjoint functor theorem|title=Adjoint functor theorem}}
* {{nlab|id=solution set condition|title=Solution set condition}}
* {{웹 인용|url=https://unapologetic.wordpress.com/2007/07/16/adjoint-functors/|제목=Adjoint functors|날짜=2007-07-16|웹사이트=The Unapologetic Mathematician|이름=John|성=Armstrong|언어=en}}
* {{웹 인용|url=http://mathoverflow.net/questions/5786/how-do-i-check-if-a-functor-has-a-left-right-adjoint|제목=How do I check if a functor has a (left/right) adjoint?|웹사이트=Math Overflow}}
* {{웹 인용|url=https://unapologetic.wordpress.com/2007/07/17/the-unit-and-counit-of-an-adjunction/|제목=The unit and counit of an adjunction|날짜=2007-07-17|웹사이트=The Unapologetic Mathematician|이름=John|성=Armstrong|언어=en}}
* {{웹 인용|url=http://mathoverflow.net/questions/5786/how-do-i-check-if-a-functor-has-a-left-right-adjoint|제목=How do I check if a functor has a (left/right) adjoint?|웹사이트=Math Overflow|언어=en}}
 
== 같이 보기 ==